PLEASE HELPPP ME MATH

PLEASE HELPPP ME MATH

Answers

Answer 1

The coordinates of the image of point D that ensures that the triangles are congruent is (-2, 2)

How to determine the coordinates of point D

Given the triangle ABC and the incomplete triangle DEF

From the triangle transformation of points B and C, we can see that

The points of the triangle ABC are translated to the left by 6 units and downward by 4 units

Mathematically, this can be represented as

(x, y) = (x - 6, y - 4)

Given that

A = (4, 6)

So, we have

D = (4 - 6, 6 - 4)

Evaluate the difference

D = (-2, 2)

Hence, the position is (-2, 2)

Read more about transformation at

https://brainly.com/question/27224272

#SPJ1


Related Questions

Find the Z-score for each of the following IQ scores
90 160(Einstein's IQ)

Answers

Answer:

z=3.75

Step-by-step explanation:

MOM+DAD=FAST DAD is a multiple of 83 cryptarithm

Answers

Answer:1+1=3

Step-by-step explanation: if 1 doesn't use protection the 1 and the other 1 are going to make 3

1. In the figure below, solve for the missing side (seems harder then it should be)
A.7
B.8
C.9
D.10

Answers

Answer:

1. a square plus b square is c square where c is the hypotenuse.

so 1. 7^2 + 6^2 = c^2

49+36=c^2

85=c^2

the square root of 85 is c

9.22 is c

all the other questions are blurry :OO

Step-by-step explanation:

A company rents storage sheds shaped like rectangular prisms. Each shed is 11 feet long, 7 feet wide, and 12 feet tall. The rental cost is $3 per cubic foot. How much does it cost to rent one shed?

Answers

The cost to rent one shed of the rectangular prism shaped shed is $2772.

What is area?

The size of a section on a surface is determined by its area. Surface area refers to the area of an open surface or the border of a three-dimensional object, whereas the area of a plane region or plane area refers to the area of a shape or planar lamina.

What is a prism?

A rectangular prism is a polyhedron in geometry that has two parallel and congruent sides. It also goes by the name cuboid. Six faces, each with a rectangle form and twelve edges, make up a rectangular prism. It is referred to as a prism because of the extent of its cross-section.

Volume of prism= BH

where B= area of base and H= height

B= 11*7 = 77 feet²

H= 12 feet

Volume= 77*12=924 cubic feet

Cost =$3 per cubic foot

Total cost= 3*924= $2772

To know more about area, visit

https://brainly.com/question/27683633

#SPJ1

HELP ASAP PLEASE! A yard plan includes a rectangular garden that is surrounded by bricks. In the drawing, the garden is 7 inches by 4 inches. The length and width of the actual garden will be 35 times larger than the length and width in the drawing.

What is the perimeter of the drawing? Show your work.

What is the perimeter of the actual garden? Show your work.

What is the effect on the perimeter of the garden with the dimensions are multiplied by 35? Show your work.

Answers

The perimeter of the garden in the drawing is 22 inches and the perimeter of the actual garden is 770 inches

The perimeter of the drawing

The perimeter of the garden in the drawing is calculated as

Perimeter of garden in drawing = 2(length + width)

So, we have

Perimeter of garden in drawing = 2(7 inches + 4 inches)

Evaluate

Perimeter of garden in drawing = 22 inches

So the perimeter of the garden in the drawing is 22 inches.

The perimeter of the actual garden

For the perimeter of the actual garden, we have

Perimeter of actual garden = 22 inches * 35

Perimeter of actual garden = 770 inches

The effect on the perimeter

We can see that the perimeter of the actual garden is 35 times larger than the perimeter of the garden in the drawing.

This makes sense since the length and width of the actual garden are 35 times larger than the length and width in the drawing, so the perimeter (which is the sum of the lengths of all four sides) would also be 35 times larger.

Read more about scale drawing at

https://brainly.com/question/29229124

#SPJ1

Total of 18 students 5 students prefer country. what is the probability?

Answers

Answer:

27.78%

Step-by-step explanation:

We know

Total of 18 students 5 students prefer country.

What is the probability?

5/18 = 27.78%

So, the answer is 27.78%

Assuming boys and girls are equally likely, find the probability of a couple's last child being a baby boy. They have a total of 3 children not including the last child in question and all were boys before the last child was born.
Express your answer as a percentage rounded to the nearest hundredth without the % sign.

Answers

Answer:

Step-by-step explanation:

The probability of a baby being a boy or a girl is 1/2 or 50% each. Since the couple has already had three boys, the probability of the fourth child being a boy or a girl is still 1/2 or 50%, as the gender of each child is independent of the others. Therefore, the probability of the couple's last child being a baby boy is 50%.

The required probability of a couple having a baby boy when their third child is​ born is 1/2 / 50%.

What is probability?

probability is the ratio of the number of favorable outcomes and the total number of possible outcomes. The chance that a particular event (or set of events) will occur expressed on a linear scale from 0 (impossibility) to 1 (certainty), also expressed as a percentage between 0 and 100%.

Given:

Assuming boys and girls are equally​ likely.

The first two children were both boys

According to given question we have

The probability of having a baby girl is an independent probability.

The first two children were both boys

So, it is not related to the previous child.

So required probability = 1/2 / 50%

Therefore, the required probability of a couple having a baby boy when their third child is​ born is 1/2 / 50%.

Learn more details about probability here:

https://brainly.com/question/11234923

10% of the cars in my neighborhood are red, and the rest of the cars in the neighborhood are silver. We'll call "seeing a red car" a success, and "seeing a silver car" a failure for the purposes of this problem.

Suppose that I watch 3 cars pass my house and that I become interested in the probability that exactly one of the three cars is red.

Apply the binomial formula to find the probability that exactly one of the three cars is red. Be sure to clearly state the values of n, x, and p in this case.

Answers

Answer:

In this scenario, we have:

n = 3 (since we are watching 3 cars)

x = 1 (since we are interested in the probability of exactly one car being red)

p = 0.1 (since the probability of a car being red is 10%, or 0.1)

The binomial formula for calculating the probability of exactly x successes in n independent trials with a probability of success p is:

P(x) = (nCx) * p^x * (1-p)^(n-x)

where nCx is the binomial coefficient, which can be calculated as:

nCx = n! / (x! * (n-x)!)

Using these values and the binomial formula, we can calculate the probability of exactly one of the three cars being red as:

P(1) = (3C1) * 0.1^1 * (1-0.1)^(3-1)

= (3) * 0.1 * 0.81

= 0.243

Therefore, the probability of exactly one of the three cars being red is 0.243.

In this problem, we want to find the probability that exactly one of the three cars is red, given that 10% of the cars in the neighborhood are red and we watch 3 cars pass by.

Let's define the following variables:

n = 3 (the number of trials or cars we observe)
x = 1 (the number of successes we want, i.e. seeing one red car)
p = 0.1 (the probability of success, i.e. seeing a red car)

Using the binomial formula, the probability of getting exactly one red car in three cars passing by is:

P(x = 1) = (n choose x) * p^x * (1 - p)^(n - x)

where (n choose x) is the binomial coefficient, which represents the number of ways to choose x objects from a set of n objects, and is calculated as:

(n choose x) = n! / (x! * (n - x)!)

Plugging in the values for n, x, and p, we get:

P(x = 1) = (3 choose 1) * 0.1^1 * (1 - 0.1)^(3 - 1)
= 3 * 0.1 * 0.81
= 0.243

Therefore, the probability that exactly one of the three cars is red is 0.243 or approximately 24.3%.

Question 4 X Suppose that starting today, you make deposits at the beginning of each quarterly period for the next 40 years. The first deposit is for 400, but you decrease the size of each deposit by 1% from the previous deposit. Using an nominal annual interest rate of 8% compounded quarterly, find the future value (i.e. the value at the end of 40 years) of these deposits. Give your answer as a decimal rounded to two places (i.e. X.XX).

Answers

if we make quarterly deposits and invest them at an nominal annual interest rate of 8% compounded quarterly for 40 years, we will have $143,004.54 at the end of the 40 years.

The first step in solving this problem is to calculate the amount of each quarterly deposit. We know that the first deposit is $400, and each subsequent deposit decreases by 1% from the previous deposit. This means that each deposit is 99% of the previous deposit. To calculate the size of each deposit, we can use the following formula:

deposit_ n = deposit_(n-1) * 0.99

Using this formula, we can calculate the size of each quarterly deposit as follows:

deposit_1 = $400

deposit_2 = deposit_1 * 0.99 = $396.00

deposit_3 = deposit_2 * 0.99 = $392.04

deposit_4 = deposit_3 * 0.99 = $388.12

...

We can continue this pattern for 40 years (160 quarters) to find the size of each quarterly deposit.

Next, we need to calculate the future value of these deposits using an nominal annual interest rate of 8% compounded quarterly. We can use the formula for compound interest to calculate the future value:

[tex]FV = PV * (1 + r/n)^(n*t)[/tex]

where FV is the future value, PV is the present value (which is zero since we are starting with deposits), r is the nominal annual interest rate (8%), n is the number of times the interest is compounded per year (4 since we are compounding quarterly), and t is the number of years (40).

We can substitute the values into the formula and solve for FV:

[tex]FV = $400 * (1 + 0.08/4)^(440) + $396.00 * (1 + 0.08/4)^(439) + $392.04 * (1 + 0.08/4)^(4*38) + ... + $1.64 * (1 + 0.08/4)^4[/tex]

After solving this equation, we get a future value of $143,004.54, rounded to two decimal places. This means that if we make quarterly deposits and invest them at an nominal annual interest rate of 8% compounded quarterly for 40 years, we will have $143,004.54 at the end of the 40 years.

This calculation highlights the power of compound interest over long periods of time. By making regular contributions and earning interest on those contributions, our investment grows exponentially over time. It also shows the importance of starting early and consistently contributing to an investment over time in order to achieve long-term financial goals.

To know more about quarterly deposit.  click here:

brainly.com/question/17218467

#SPJ4

A local hamburger shop sold a combined total of 688 hamburgers and cheeseburgers on Monday. There were 62 fewer cheeseburgers sold than hamburgers. How many hamburgers were sold on Monday? I hamburgers​

Answers

Answer: 375 hamburgers

Step-by-step explanation:

Let's assume that x is the number of hamburgers sold on Monday.

According to the problem, the number of cheeseburgers sold is 62 fewer than the number of hamburgers sold. So the number of cheeseburgers sold is x-62.

The total number of hamburgers and cheeseburgers sold is 688. So we can set up an equation:

x + (x-62) = 688

Simplifying this equation, we get:

2x - 62 = 688

Adding 62 to both sides, we get:

2x = 750

Dividing both sides by 2, we get:

x = 375

Therefore, 375 hamburgers were sold on Monday.

307 cheeseburgers and 369 hamburgers.

Help pleaseeeeeeeeee!

Answers

Answer:

y = 0.8x

Step-by-step explanation:

machine fills 24 jars in 30 seconds , then

fills 1 jar in [tex]\frac{24}{30}[/tex] = 0.8 seconds

thus number of jars filled in x seconds is

y = 0.8x

In a group of rectangles, the length of each rectangle is twice the width. Is this an additive or multiplicative relationship? Explain your reasoning.​

Answers

Answer:

Multiplicative

Step-by-step explanation:

If the area of the first rectangle is l*w, the area of the second would be 2w*w. The area is multiplied by 2 and the relationship is therefore multiplicative.

PLEASE ASAP!!Graph the line 4x+5y=20

Answers

Step-by-step explanation:

Might be a little easier to visualize if yo re-arrange it into y = mx + b form:

4x+ 5y = 20

5y = -4x + 20

y = - 4/5 x + 4         y-axis intercept at y = 4

                                x axis intercept is found by:

                                     0 = -4/5 x + 4

                                             - 4 = -4/5 x

                                                 x = 5

So===> plot the two intercept points ( 0,4)  and  ( 5,0)  and connect the dots

a
21 units squared
b
27.6 units squared
c
32.2 units squared
d
42 units squared

Answers

The area of the right triangle given in this problem is given as follows:

21 units squared -> Option A.

How to obtain the area of a triangle?

To calculate the area of a triangle, you can use the formula presented as follows:

Area = (1/2) x base x height

In which the parameters are given as follows:

"base" is the length of the side of the triangle that is perpendicular to the height."height" is the length of the perpendicular line segment from the base to the opposite vertex.

For a right triangle, we can consider one side to be the base and the other side to be the height, hence the parameters are given as follows:

Base of 7 units.Height of 6 units.

Hence the area of the triangle is given as follows:

A = 0.5 x 7 x 6 = 21 units squared.

Missing Information

The complete problem is defined as follows:

"Calculate the area of the given triangle".

More can be learned about the area of a triangle at brainly.com/question/21735282

#SPJ1

Consider the system described below with input f(t) and output y(t). Determine if the system is linear or nonlinear. Show all work. dy +3 +3ty(t)=1² f(t) dt 5. By direct integration find the Laplace transform of the signal shown. f (t) 1+6) t(s)

Answers

The system described above is nonlinear because it contains a term with y(t) multiplied by t. The Laplace transform of f(t) is (6/s²)+(1/s).

If we substitute y1(t) and y2(t) into the equation and add them together, we get:

dy1/dt + 3 + 3ty1(t) = 1² f(t) dt dy2/dt + 3 + 3ty2(t) = 1² f(t) dt

Then we can add these two equations together to get:

d(y1+y2)/dt + 3 + 3t*(y1+y2)(t) = 2*1² f(t) dt

This is not equal to the original equation with y(t), which means that the system is nonlinear.

To find the Laplace transform of f(t), we can use the formula:

L{f(at+b)} = (1/a) ×F(s-b/a)

where F(s) is the Laplace transform of f(t). In this case, we have:

f(t) = (1+6t)

So we can rewrite this as:

f(t) = (6×t+1)

Now we can use the formula to find the Laplace transform:

L{(6×t+1)} = (6/s²)+(1/s)

Therefore, the Laplace transform of f(t) is (6/s²)+(1/s).

To know more about Laplace transform, refer:

https://brainly.com/question/17062586

#SPJ4

5/9=

1/14=

12/13=

2/13=

9/11=

9/17=
To round each fraction

Answers

Answer:

Step-by-step explanation:

1. Rounded to 0.56

2. Rounded

Question 2
State the probability that a randomly selected, normally
distributed value lies between
a) o below the mean and o above the mean (round to
the nearest hundredths)
b) 20 below the mean and 20 above the mean (round
to the nearest hundredths)

Answers

The probability of a randomly selected, normally distributed value lying between 20 below the mean and 20 above the mean is 0.9545 (rounded to the nearest hundredth).

What is the fundamental concept of probability?

A number between zero and one represents the probability that an occurrence will take place. An event is a predefined set of random variable outcomes. Only one mutually exclusive event can occur at a time. Exhaustive events encompass or include all possible outcomes.

We can calculate the probabilities of a randomly chosen value falling between different z-scores using the provided standard normal distribution table.

a) The probability of a value being 0 below or above the mean is the same as the probability of a value being -1 to 1 standard deviations from the mean. According to the standard normal distribution table, the probability of a z-score between -1 and 1 is 0.6827. As a result, the probability of a randomly chosen, normally distributed value falling between 0 below and 0 above the mean is 0.6827. (rounded to the nearest hundredth).

b) The probability of a value falling between 20 and 20 standard deviations from the mean is the same as the probability of a value falling between -20/10 and 20/10 standard deviations from the mean (since the standard deviation is 10). According to the standard normal distribution table, the probability of a z-score between -2 and 2 is 0.9545. As a result, the probability of a randomly chosen, normally distributed value falling between 20 below and 20 above the mean is 0.9545. (rounded to the nearest hundredth).

To know more about probability visit:

https://brainly.com/question/30719832

#SPJ1

LetY1,Y2,Yn denote a random sample of size n from a population whose density is given by f(y)={αyα−1θα,0≤y≤θ0,elsewhere,where α>0 is a known, fixed value, but θ is unknown. Consider the estimator ˆθ=max(Y1,Y2,...Yn).(a) Show that ˆθ is a biased estimator for θ.(b) Find a multiple of ˆθ that is an unbiased estimator of θ.(c) Derive MSE(ˆθ).

Answers

(a) θ is a biased estimator for θ.

(b) (α+1)Y/α is an unbiased estimator of θ.

(c) MSE(θ) = αθ^2/[(α+1)^2(α+2)]

(a) To show that θ is a biased estimator for θ, we need to show that E(θ) ≠ θ.

Using the definition of the maximum function, we have

P(θ ≤ y) = P(Y1 ≤ y, Y2 ≤ y, ..., Yn ≤ y) = (F(y))^n

where F(y) is the cumulative distribution function of Y.

Differentiating both sides with respect to y, we get:

f(θ) = n(F(θ))^(n-1)f(θ)

Simplifying, we get

F(θ) = (1/n)^(1/(n-1))

Using this result, we can find the expected value of θ

E(θ) = ∫₀^∞ θf(θ)dθ = ∫₀^θ θαθ^α-1dθ = αθ/(α+1)

Thus, E(θ) ≠ θ, which means that θ is a biased estimator for θ.

(b) To find a multiple of θ that is an unbiased estimator of θ, we can use the method of moments.

We know that the population mean of Y is

μ = ∫₀^θ yf(y)dy = αθ/(α+1)

The sample mean is

Y = (Y1+Y2+...+Yn)/n

Equating these two expressions and solving for θ, we get

θ = (α+1)Y/α

Thus, (α+1)Y/α is an unbiased estimator of θ.

(c) The mean squared error (MSE) of θ can be written as

MSE(θ) = E[(θ - θ)^2]

Expanding the square and using the linearity of expectation, we have

MSE(θ) = E[θ^2] - 2θE[θ] + E[θ]^2

We already know that E[θ] = αθ/(α+1).

To find E[θ^2], we can use the fact that θ = max(Y1,Y2,...Yn)

P(θ ≤ y) = P(Y1 ≤ y, Y2 ≤ y, ..., Yn ≤ y) = (F(y))^n

Differentiating both sides with respect to y, we get

f(θ) = n(F(θ))^(n-1)f(θ)

Using this result, we can find E[θ^2]

E[θ^2] = ∫₀^∞ θ^2f(θ)dθ = ∫₀^θ θ^2αθ^α-1dθ = αθ^2/(α+2)

Substituting these expressions into the MSE formula, we get

MSE(θ) = αθ^2/(α+2) - 2θ(αθ/(α+1)) + (αθ/(α+1))^2

Simplifying, we get

MSE(θ) = αθ^2/[(α+1)^2(α+2)]

Learn more about biased estimator here

brainly.com/question/30982574

#SPJ4

Shallow Drilling, Inc. has 76,650 shares of common stock outstanding with a beta of 1.47 and a market price of $50.00 per share. There are 14,250 shares of 6.40% preferred stock outstanding with a stated value of $100 per share and a market value of $80.00 per share. The company has 6,380 bonds outstanding that mature in 14 years. Each bond has a face value of $1,000, an 8.00% semiannual coupon rate, and is selling for 99.10% of par. The market risk premium is 9.79%, T-Bills are yielding 3.21%, and the tax rate is 26%. What discount rate should the firm apply to a new project's cash flows if the project has the same risk as the company's typical project?
Group of answer choices

Answers

The discount rate that should be applied to a new project's cash flows is the Weighted Average Cost of Capital (WACC). To calculate WACC, you need to first calculate the cost of debt. This is done by taking the face value of the bonds ($1000) multiplied by the coupon rate (8%) multiplied by (1 - the tax rate (26%)), which equals 5.92%. The cost of debt is then calculated by taking the market value of the debt (6,380 x $1,000 x 99.1%) and dividing this by the total market value of the debt plus the market value of the equity (6,380 x $1,000 x 99.1% + 76,650 x $50 + 14,250 x $80), which equals 5.22%.

Next, you need to calculate the cost of equity using the Capital Asset Pricing Model (CAPM). This is done by taking the risk-free rate (3.21%) plus the market risk premium (9.79%) multiplied by the firm's beta (1.47), which equals 17.18%.

The WACC is then calculated by taking the cost of equity multiplied by the proportion of equity (76,650 x $50 + 14,250 x $80 divided by the total market value of the debt plus the market value of the equity) plus the cost of debt multiplied by the proportion of debt (6,380 x $1,000

3 Tell whether each statement is True or False.
A 20° angle and a 70° angle can be
composed into a 90° angle.
b. Three 50° angles compose an angle
that measures 350⁰.
c. A 15° angle and a 60° angle compose
an angle that measures 75°.
d. Four 50° angles can be composed
into a 200° angle.
True False
True
True
True
4 Look at the drawing of a hand fan at the right. The
False
False
Fal

Answers

d. True. The sum of four 50° angles is 200°, so they can compose a 200° angle.

what is a geometric sequence?

A geometric sequence is a sequence of numbers in which each term after the first is found by multiplying the previous term by a fixed constant called the common ratio. The common ratio is denoted by the letter r.

The general formula for a geometric sequence is:a₁, a₁r, a₁r², a₁r³, ...

a. False. The sum of a 20° angle and a 70° angle is 90°, so they can compose a 90° angle.

b. False. The sum of three 50° angles is 150°, so they cannot compose an angle that measures 350⁰.

c. False. The sum of a 15° angle and a 60° angle is 75°, so they can compose an angle that measures 75°.

Therefore, d. True. The sum of four 50° angles is 200°, so they can compose a 200° angle.

To learn more geometric sequence from the given link:

https://brainly.com/question/16274868

#SPJ1

Please help me solve this my head hurts

Answers

a. Nοne οf these measure οf central tendency dοn't exist fοr this data set. Optiοn d) is cοrrect

b. As the sum οf all οbservatiοns wοuld chance, the mean wοuld be affected by the change. Optiοn a) is cοrrect

What is central tendency?

In statistics, the central tendency is the descriptive summary οf a data set. Thrοugh the single value frοm the dataset, it reflects the centre οf the data distributiοn. Mοreοver, it dοes nοt prοvide infοrmatiοn regarding individual data frοm the dataset, where it gives a summary οf the dataset. Generally, the central tendency οf a dataset can be defined using sοme οf the measures in statistic.

c.

Suppοse that, the largest measurement 97 is remοved.

The number οf οbservatiοns as well as the sum οf all οbservatiοns wοuld change.

Therefοre, the median and the mean wοuld be changed and οptiοn a) and b) are cοrrect.

d.

Since there are three mοdes, the mean, median and mοde must nοt be cοmpared tο each οther fοr skewness.

Instead, it is required tο grοup data in intervals and οbserve the pattern οf classes versus frequencies, as displayed in histοgram.

Therefοre, the distributiοn appears rοughly symmetric and οptiοn c) is cοrrect.

Learn more about central tendency

https://brainly.com/question/30218735

#SPJ1

. If h> 3 and h - 2g= 0, which of
the following must be true?
A. g> 2.5
B. g> 1.5
C. g <0.5
D. g <1.5
E. g>2

Answers

By linear equality , g >1.5 is must be true.

What are equality and inequality along a line?

Equal (=) is the symbol used in linear equations. Example. Using the inequality symbols (>,, is greater than or equal to, and is less than or equal to), linear inequalities are expressed.

                          x - 5 > 3x - 10 is an illustration of a linear inequality. As the larger than symbol is employed in this inequality, the LHS is strictly greater than the RHS. After being solved, the inequality appears as 2x 5 x (5/2).

If h> 3 and h - 2g= 0

H=2g

2g>3

g >1.5

Learn more about linear equality

brainly.com/question/11874554

#SPJ1

Which answer choice contains all the factors of 10? • A. 1, 2, 5 О B. 1, 2, 5, 10 O C. 2, 5 O D. 1, 10

Answers

Answer:

The answer would be B (1,2,5, 10)

Step-by-step explanation:

Since one can already go into any number that greater than zero it would be a factor

2x5=10 as well so two and five would be a factor

you can also do 10x1

or 1x10 so both 10 and one would be factors of ten.

Determine whether each of these functions from
Z
to
Z
is one-to-one (i.e., injective). (3 pts in total) (a)
f(n)=n−3
(b)
f(n)=n 2
−1
(c)
f(n)=n 5

Answers

The function f(n) = n - 3 maps distinct integers to distinct integers, and thus is injective.

(a) The function f(n) = n - 3 is one-to-one (injective). To prove this, suppose that f(a) = f(b) for some integers a and b. Then, we have a - 3 = b - 3, which implies a = b. Therefore, the function f(n) = n - 3 maps distinct integers to distinct integers, and thus is injective.

(b) The function f(n) = n^2 - 1 is not one-to-one (not injective). To see this, note that f(1) = f(-1) = 0, so different inputs map to the same output. In general, for any positive integer k, we have f(k) = f(-k), since (k^2 - 1) = ((-k)^2 - 1). Therefore, the function f(n) = n^2 - 1 is not injective.

(c) The function f(n) = n^5 is one-to-one (injective). To prove this, suppose that f(a) = f(b) for some integers a and b. Then, we have a^5 = b^5, which implies a = b (since the fifth root of a non-zero real number is unique). Therefore, the function f(n) = n^5 maps distinct integers to distinct integers, and thus is injective.

Find out more about functionsat  

brainly.com/question/13262879

#SPJ4

Question 20
There were 750 shirts in a box. 20% of them were pink and 18% were green. The
remaining shirts were either yellow or black. If there were 85 more black shirts than
yellow shirts, what was the total number of black and green shirts in the box?

Answers

Answer:

410 Black and Green shirts

Step-by-step explanation:

150 shirts were pink

135 shirts were green

so knowing this we can set up the equation 2x+85=465

there were 275 black shirts in the box

and 190 yellow shirts

therefore there were 410 Black and Green shirts in the box

(4+4i)(5-i) perform the indicated operation and express the result as a simplified complex number

Answers

Answer:

24 + 16i

Step-by-step explanation:

[tex](4+4i)(5-i)\\=(4)(5)+(4)(-i)+(4i)(5)+(4i)(-i)\\=20-4i+20i-4i^2\\=20+16i-4(-1)\\=20+16i+4\\=24+16i[/tex]

Solve the following problems.
Given: AABC, DE AC,
BD DC, mZ1=m22,
mZBDC= 100°
Find: m< A, m< b , m

Answers

The value of the angles in the triangle are:

∠A = 60°, ∠B = 80° and ∠C = 40°

How to find the value of m∠A, m∠B, m∠C in the triangle?

We  are given that BD = DC

Thus, ∠DBC = ∠BCD ---- 1  (angle in isosceles triangle)

We also have ∠BDC = 100°

In ΔBDC

∠BDC + ∠DBC + ∠BCD = 180°  (sum of angles of triangle is 180°)

Using 1:

∠BDC + 2∠DBC = 180°  

100° + 2∠DBC = 180°  

2∠DBC = 180 - 100

2∠DBC = 80

∠DBC = 80/2

∠DBC = 40°

∠DBC = ∠BCD = ∠2 = 40°

Thus, ∠C = 40°

We are given that  m∠1 = m∠2

Thus, ∠1 = ∠2 = 40°

Now,  ∠BDC +  ∠BDA = 180° (Linear pair)

100° + ∠BDA = 180°

∠BDA = 180 - 100

∠BDA = 80°

In ΔABD

∠ABD + ∠BDA +  ∠BAD = 180° (sum of angles of triangle is 180°)

∠1 + ∠BDA +  ∠BAD = 180°

40° +  80° +  ∠BAD = 180°

120° +  ∠BAD = 180°

∠BAD = 60°

So,  ∠A = 60°

∠B =  ∠1  + ∠2 = 40° + 40° = 80°

Therefore,  ∠A = 60°, ∠B = 80° and ∠C = 40°

Learn more about triangle on:

brainly.com/question/9132922

#SPJ1

Complete Question

m∠1=m∠2

D∈

AC

, BD = DC

m∠BDC = 100°

Find: m∠A, m∠B, m∠C

Individuals who identify as male and female were surveyed
regarding their diets.
Meat-
eater
Male 35
Female 37
Total 72
.
Vegetarian Pescatarian Vegan Total
12
23
35
24
14
38
18
27
45
89
101
190
What is the probability that a randomly selected
person is a pescatarian or female? Round your
answer to the hundredths place.

Answers

Answer:

If you add all the numbers together and divide the number of females by the total number of people. It is a 5% chance that out of all the people, the group of 37 females, one would be selected.

If you add up all the numbers of people and divide by the number of pescatarians, there is an 89.5% chance of a pescatarian being selected.

all numbers added together, 72 meat eaters, and 616 pescatarians = 688

females = 37

37/688 = 0.0537 = 5.37 = rounded to hundredths place = 5%

all numbers added together = 688

number of pescatarians = 616

616/688 = .0895 = 89.5%

Step-by-step explanation:

- Please help me, I don't understand

What is the specific heat of an unknown substance if 100.0 g of it at 200.0 °C reaches an equilibrium temperature of 27.1 °C when it comes in contact with a calorimeter of water. The water weighs 75. g and had an initial temperature of 20.00 °C? (Specific heat of water is 4.18 J/g°C). Show your work

Answers

Answer:The specific heat of a substance is defined as the amount of heat required to raise the temperature of one gram of the substance by one degree Celsius (or Kelvin).

To find the specific heat of the unknown substance, we can use the following equation:

Q = m x c x ΔT

where Q is the heat gained or lost, m is the mass of the substance, c is its specific heat, and ΔT is the change in temperature.

In this problem, we know the mass and initial and final temperatures of both the unknown substance and the water, as well as the specific heat of water. We can use this information to calculate the heat gained by the water, which must be equal to the heat lost by the unknown substance:

Heat gained by water = Heat lost by unknown substance

m(water) x c(water) x ΔT(water) = m(substance) x c(substance) x ΔT(substance)

We can plug in the values we know and solve for the specific heat of the unknown substance:

m(water) = 75.0 g

c(water) = 4.18 J/g°C

ΔT(water) = 27.1 °C - 20.00 °C = 7.1 °C

m(substance) = 100.0 g

ΔT(substance) = 200.0 °C - 27.1 °C = 172.9 °C

75.0 g x 4.18 J/g°C x 7.1 °C = 100.0 g x c(substance) x 172.9 °C

Simplifying this equation, we get:

c(substance) = (75.0 g x 4.18 J/g°C x 7.1 °C) / (100.0 g x 172.9 °C)

c(substance) = 0.197 J/g°C

Therefore, the specific heat of the unknown substance is 0.197 J/g°C.

Step-by-step explanation:

Answer:

The specific heat of the unknown substance is 0.39 J/g°C.

Step-by-step explanation:

To solve this problem, we can use the principle of conservation of energy, which states that the heat lost by the unknown substance is equal to the heat gained by the water and the calorimeter. We can express this principle mathematically as:

Q_lost = Q_gained

where Q_lost is the heat lost by the unknown substance, and Q_gained is the heat gained by the water and calorimeter.

We can calculate Q_lost using the formula:

Q_lost = m × c × ΔT

where m is the mass of the unknown substance, c is its specific heat, and ΔT is the change in temperature it undergoes.

We can calculate Q_gained using the formula:

Q_gained = (m_water + m_calorimeter) × c_water × ΔT

where m_water is the mass of the water, m_calorimeter is the mass of the calorimeter, c_water is the specific heat of water, and ΔT is the change in temperature of the water and calorimeter.

Since the system reaches an equilibrium temperature, we can set Q_lost equal to Q_gained and solve for the specific heat of the unknown substance (c).

Here's the calculation:

Q_lost = Q_gained

m × c × ΔT = (m_water + m_calorimeter) × c_water × ΔT

100.0 g × c × (200.0 °C - 27.1 °C) = (75.0 g + 75.0 g) × 4.18 J/g°C × (27.1 °C - 20.00 °C)

Simplifying:

c = [(75.0 g + 75.0 g) × 4.18 J/g°C × (27.1 °C - 20.00 °C)] / (100.0 g × (200.0 °C - 27.1 °C))

c = 0.39 J/g°C

Therefore, the specific heat of the unknown substance is 0.39 J/g°C.

PLEASE HELP MEE with all four questionsss

Answers

Therefore, the distance between the 90 degree angle and the hypotenuse is approximately 0.829 units.

What is triangle?

A triangle is a two-dimensional geometric shape that is formed by three straight line segments that connect to form three angles. It is one of the most basic shapes in geometry and has a wide range of applications in mathematics, science, engineering, and everyday life. Triangles can be classified by the length of their sides (equilateral, isosceles, or scalene) and by the size of their angles (acute, right, or obtuse). The study of triangles is an important part of geometry, and their properties and relationships are used in many areas of mathematics and science.

Here,

1. To find HF, we can use the angle bisector theorem, which states that if a line bisects an angle of a triangle, then it divides the opposite side into two segments that are proportional to the adjacent sides. Let's denote the length of HF as x. Then, by the angle bisector theorem, we have:

JF/FH = JG/HG

Substituting the given values, we get:

15/x = 18/21

Simplifying and solving for x, we get:

x = 15 * 21 / 18

x = 17.5

Therefore, HF is 17.5 cm.

2. Let's denote the length of the hypotenuse as h and the length of the leg opposite the 18-unit perpendicular as a. We can then use the Pythagorean theorem to write:

h² = a²  + 18²

We are told that the hypotenuse is divided into segments of length x and 6 units, so we can write:

h = x + 6

Substituting this expression into the first equation, we get:

(x + 6)² = a² + 18²

We are also told that the leg adjacent to the angle opposite the 4-unit segment is divided into segments of length 4 and (a - 4), so we can write:

a = 4 + (a - 4)

Simplifying this equation, we get:

a = a

Now we can substitute this expression for a into the previous equation and solve for x:

(x + 6)² = (4 + (a - 4))² + 18²

Expanding and simplifying, we get:

x² + 12x - 36 = 0

Using the quadratic formula, we get:

x = (-12 ± √(12² - 4(1)(-36))) / (2(1))

x = (-12 ± √(288)) / 2

x = -6 ± 6√(2)

Since the length of a segment cannot be negative, we take the positive root:

x = -6 + 6sqrt(2)

x ≈ 1.46

Therefore, the value of x is approximately 1.46 units.

3. Let's denote the length of the hypotenuse as h and the length of the leg adjacent to the angle opposite the 9-unit perpendicular as b. We can then use the Pythagorean theorem to write:

h² = b² + 9²

We are told that the hypotenuse is divided into segments of length x and 6 units, so we can write:

h = x + 6

Substituting this expression into the first equation, we get:

(x + 6)² = b² + 9²

Expanding and simplifying, we get:

x² + 12x - b² = 27

We also know that the length of the leg opposite the 9-unit perpendicular is:

a = √(h² - 9²)

= √((x + 6)² - 9²)

= √(x² + 12x + 27)

Now we can use the fact that the tangent of the angle opposite the 9-unit perpendicular is equal to the ratio of the lengths of the opposite and adjacent sides:

tan(θ) = a / b

Substituting the expressions for a and b, we get:

tan(θ) = √(x² + 12x + 27) / (x + 6)

We also know that the tangent of the angle theta is equal to the ratio of the length of the opposite side to the length of the adjacent side:

tan(θ) = 9 / b

Substituting the expression for b, we get:

tan(θ) = 9 / √(h² - 9²)

Substituting the expression for h, we get:

tan(θ) = 9 / √((x + 6)² - 9²)

Since the tangent function is the same for equal angles, we can set these two expressions for the tangent equal to each other:

√(x² + 12x + 27) / (x + 6) = 9 / √((x + 6)² - 9²)

Squaring both sides, we get:

(x² + 12x + 27) / (x + 6)² = 81 / ((x + 6)² - 81)

Cross-multiplying and simplifying, we get:

x⁴ + 36x³ + 297x² - 1458x - 2916 = 0

Using a numerical method such as the Newton-Raphson method or the bisection method, we can find the approximate solution to this equation:

x ≈ 9.449

Therefore, the value of x is approximately 9.449 units.

4. Let's denote the length of the hypotenuse as h and the length of the leg adjacent to the angle opposite the distance we want to find as b. We can use the Pythagorean theorem to write:

h² = b² + d²

We are told that the hypotenuse is divided into segments of length 9 and 4 units, so we can write:

h = 9 + 4 = 13

Substituting this expression into the first equation, we get:

13² = b² + d²

Simplifying and solving for d, we get:

d = √(13² - b²)

Now, we need to find the value of b. We know that the hypotenuse is divided into segments of length 9 and 4 units, so we can use similar triangles to write:

b / 4 = 9 / 13

Simplifying and solving for b, we get:

b = 36 / 13

Substituting this expression for b into the equation we found earlier for d, we get:

d = √(13² - (36/13)²)

Simplifying and finding a common denominator, we get:

d =√ (169*13 - 36²) / 13²

Simplifying further, we get:

d = √(169169 - 3636) / 169

Calculating this expression, we get:

d ≈ 0.829

To know more about triangle,

https://brainly.com/question/28600396

#SPJ1

Other Questions
Suppose that annual stock returns for a particular company are normally distributed, with a mean of 16% and a standard deviation of 10%. You are going to invest in this stock for one year.A) Find the probability that your one-year return will exceed 30%.B) Find that probability that your one-year return will be negative. Generally speaking, if you are risk averse, you should save money for retirement in a savings account rather than investing in stocks and bonds, because a savings account will keep up with inflation. True or False Choose the sentence in which the subjects and verbs have been correctly identified and they agree. What is the verb tense of the underlined word in the sentence you chose? A) At the end of the story, they was living happily ever after. B) At the end of the story, they were living happily ever after. C) At the end of the story, they were living happily ever after. D) At the end of the story, they was living happily ever after. a group of children with measles and a group of children without measles are compared; the parents of the children are asked whether their child received a measles vaccine at any point in the past. what type of study is this? how much will 10 mg lisinopril lower blood pressure when your team begins planning how to accomplish the is 4410 group project, you notice that everyone seems to be agreeing with each other. instead of being excited that the group is so cohesive, you recognize that _______ may be occurring which will damage the quality of your project in the future. write down the name of shape W an ascribed status is a. a status into which one enters voluntarily. b. one status that stands out above all others. c. the least-used status within a status set. d. a status into which one is born. a 60-m-long, 9.4-mm-diameter rope hangs freely from a ledge. the density of the rope is 55 g > m. how much work is needed to pull the entire rope to the ledge? May 9, 2 p.m. Both you and your partner go to the Limitville County Hospital to check on the passenger, who is the driver's wife, Emily Samples. You speak to the admitting doctor, Dr. Anthony Pallor, who indicates that Mrs. Samples is still in a coma, but in stable condition. You speak to the family members, offering sympathy. You also ask them a few questions. In your crime scene notebook:Determine three questions you should ask Mrs. Samples's doctor.Determine three questions you should ask the family members.May 10, 8 a.m.At your office, you use a nationwide database to conduct a search on any charges or violations regarding Peter Samples and his wife, Emily Clairmont Samples. In your crime scene notebook:Why conduct a search on both occupants criminal histories?May 10, 9:30 a.m.You are able to obtain the officers report from the interview of the driver of the white SUV, Carl Minus. In the report, Mr. Minus indicates that he did not see the accident nor did he hear it. He came upon it around 9:50 p.m. on May 8, when his headlights showed the car smashed into the tree. He immediately pulled off to the side of the road and approached the vehicle, calling out to the victims in the car. He used the flashlight on his cellphone to look into the car and could not determine if either of them were alive. He then called 911 at approximately 9:52 p.m. to report the incident. After checking with the 911 logs, you confirm the time frame given by Mr. Minus.In your crime scene notebook:Clarify why it is necessary to corroborate the report and the 911 logs.May 10, 12 p.m.You call the hospital for an update on Mrs. Samples condition and are notified that she is still in a coma.May 10, 2 p.m.You and your partner go to the impound lot to look at the Samples' car and make sure you did not miss anything. You reexamine the point of impact on the car and take measurements of the indentation damage done to the car. In your crime scene notebook:Theorize how the measurements of the indentation damage would correlate to the speed of the car.What other evidence could give an indication as to the speed of the car?May 11, 8:30 a.m.You receive the toxicology report from Temper Labs.In your crime notebook:What drug(s) were present in Mr. Samples's body that could cause impairment?For your state, look up the legal limit of alcohol allowed. Is Mr. Samples below or above the legal limit?Revise your hypothesis based on the new evidence.Write a summary report as to the probable cause of the accident. Include evidence to support your findings. In your report, mention what may hinder you from making it conclusive.Part 2: Accessthe Blood Alcohol Level (BAC) calculator.Use the calculator to find these answers:Monday: One couple at the bar drank red wine, and each had one glass between 9 and 10 p.m. What was the BAC for the 200 lb man and the 120 lb woman when they went home at midnight?Tuesday: Another night, the same couple each drank one glass of wine every hour for four hours, beginning at 8 p.m. What was the BAC of each person at the end of that night? They went home at midnight.Saturday: The same couple returned for drinks. On Saturday night, she drank two cosmopolitans and he drank four stout beers between 8 and 10 p.m. What was the BAC of each person at the end of the night? They went home at midnight.What nights should they have taken a taxi to get home?What does the statement "If you drink, drink responsibly" mean?In your crime scene notebook:Your responses to each bullet task must include one statement answering the question and justifying the answer with numerical evidence. FILL IN THE BLANK interest groups are more likely to succeed when their request has ___ salience and when it has ___ conflict a.) Determine whether potassium hydrogen tartrate (KHC4H4O6) is neutral, basic, or acidic. First, what is its Ka when it acts as an acid? The following are for the diprotic acid, H2C4H4O6: Ka1 = 1.0 x 10-3 and Ka2 = 4.6 x 10-5. b.) Second, what is its Kb when it acts as a base? c.) Finally, indicate whether the HC4H4O6- ion is neutral, basic, or acidic in solution. Competitive markets maximize the ________ consumer surplus and producer surplus. This occurs when the economy produces where _________ . The plum pudding model hypothesized by Thompson shows the scattering of electrons. When was this discovered in relation to other scientist's atomic hypotheses? After Rutherford but before Chadwick Before Bohr but after Chadwick Before Bohr and Rutherford After Rutherford but before Bohr T/F. Religious people tend to practice the religion of their choosing, but it does not affect their overall lifestyle or their daily choices. Lab StepsYou work at a computer repair store. A customer brought in a computer for repairs. The computer has the following specifications: Intel i5 8600K processor. 16 GB DDR4 memory. Three SATA hard drives, one with the operating system installed, the other two for storing data. Single SATA optical drive. You replaced the power supply in the computer, and now the computer won't boot. In this lab, your task is to diagnose and correct the problems. You will know that you have successfully corrected all the problems when: The computer recognizes all the components in the BIOS (including the correct amount of memory). The computer boots into Windows. All the drives are shown in Explorer. If you estimate the basis will be 15 over December futures at the time you purchase corn, the approximate buying price you can lock in by selling a December futures contract at $5.50 is: (a) $5.65 (b) $5.60 (c) $5.35 (d) none of the above Find the acceleration of the distance versus time function: s = 2t^2 + 2 Choose the correct order of the numbered statements to put the last four events of photosynthesis in order. 1. A photon of light strikes photosystem I and allows it to excite the electron. 2. The electron transport chain passes the electron from photosystem II to photosystem I. 3. The excited electron is passed from photosystem I to NADPH. 4. NADPH and ATP are used in the Calvin cycle to make glucose. O A. 2,1,3,4 B. 3,1,2,4 C. 1,4,3,2 D. 1,2,4,3 SUBMIT Ray has 9 identical shoe boxes like the one shown. What is the total volume of all the shoe boxes?